To LUGNET HomepageTo LUGNET News HomepageTo LUGNET Guide Homepage
 Help on Searching
 
Post new message to lugnet.off-topic.geekOpen lugnet.off-topic.geek in your NNTP NewsreaderTo LUGNET News Traffic PageSign In (Members)
 Off-Topic / Geek / 4183
4182  |  4184
Subject: 
Re: math question (or pattern... whatever...)
Newsgroups: 
lugnet.off-topic.geek
Date: 
Mon, 3 Mar 2003 23:30:44 GMT
Viewed: 
266 times
  
In lugnet.off-topic.geek, Ross Crawford writes:

<snip>

Your friend has assumed that there is a difference between the reds, and you
know which one you have seen in your hand. As there is no information in the
original problem to substantiate this assumption, you have to assume it's
not the case, so the valid combinations are in fact:

r,r
r,b
r,w
b,w

Your friend rightly discounted the b,w combination, which leaves 3, of which
yours must be one, therefore you are correct, the chances are 33%.

ROSCO

I like driving home from work--gives me time to think about stuff...

And after reading all the great replies, I'm going to propose the following
solution:

If you have a bag with 4 marbles, 2 red, one white, one blue

and you reach your hand in and grab 2 of the marbles...

you have the following possibilities in your hand

r1 r2
r1 b
r1 w
r2 b
r2 w
b w

The second you expose one of the marbles in your hand and you know it's red,
you have the following:

r(exposed) r(unexposed)
r(e) b
r(e) w

r(u) b
r(u) w
b w

and, as shown by the separation, you say "If you know one is red (exposed)
what is the chance that the other one is red (unexposed)?"

the last 3 options are completely invalid, for they don't fit the criteria
"If one red is exposed..." (r(exposed))
so we're left with the first 3, and the chance that the r(unexposed) is
combined with r(exposed) is 1 in 3...

Thanks for all the help!

Dave K



Message is in Reply To:
  Re: math question (or pattern... whatever...)
 
(...) Your friend has assumed that there is a difference between the reds, and you know which one you have seen in your hand. As there is no information in the original problem to substantiate this assumption, you have to assume it's not the case, (...) (22 years ago, 3-Mar-03, to lugnet.off-topic.geek)

61 Messages in This Thread:




















Entire Thread on One Page:
Nested:  All | Brief | Compact | Dots
Linear:  All | Brief | Compact
    

Custom Search

©2005 LUGNET. All rights reserved. - hosted by steinbruch.info GbR